Electromagnetismo relativista y fuerzas electromagnéticas sobre 2 protones

Ejemplo A y B, donde la única diferencia es la velocidad de movimiento del observador

La pregunta que tengo al respecto es cómo podemos obtener el mismo resultado de la fuerza neta que actúa sobre los protones individuales si juzgamos el sistema desde 2 marcos de referencia diferentes. Uno usando más la parte magnética del electromagnetismo, otro más la parte eléctrica para calcular las fuerzas.

Así que trataré de explicar mi comprensión a través de estos dos ejemplos en los que obtengo diferentes resultados de aceleración de partículas dependiendo de los diferentes marcos de referencia nombrados en la Imagen como "marcos de referencia del observador".

Para simplificar, he decidido ignorar el campo magnético creado por los giros de las partículas. Creo que no importan para este ejercicio mental.

En el ejemplo A, ambos protones viajan en la misma dirección y con la MISMA velocidad. Si el observador está estacionario, ve que ambos se alejan de él a la misma velocidad. Debido a que los protones tienen la misma carga, son repelidos entre sí por la fuerza electrostática. Pero debido a que se están moviendo, el protón superior crea un campo magnético a su alrededor y ese campo actúa sobre el otro protón de tal manera que lo empuja hacia arriba (hacia el otro protón). Lo mismo es cierto para el otro protón que atrae magnéticamente al otro protón. La dirección de la fuerza neta depende de las velocidades de los protones y de la distancia entre ellos. Entonces, en un caso, pueden atraerse entre sí (altas velocidades) y, en otro caso, repelerse (juntos).

Ahora veamos el ejemplo B. El observador viaja a la misma velocidad que los protones y en la misma dirección. Los protones según los observadores no se mueven. Entonces la fuerza magnética no aparece entre los protones. Lo único que sienten los protones según el observador es la fuerza electrostática repulsiva entre ellos, por lo que los protones, sin importar qué tan lejos estén o a qué velocidad viajen (según el observador estacionario del caso A) siempre repelerse entre si.

Ahora bien, esto se contradice por completo. ¿Tiene esto algo que ver con los efectos de la dilatación del tiempo y la contracción del espacio? Y si es así, ¿cómo?

He leído los ejemplos de un cable conductor de corriente y su efecto sobre una carga independiente en movimiento. Esa imagen tiene sentido debido a la contracción del espacio, por lo que hay más cargas opuestas (estacionarias) en el mismo volumen de espacio (en un cable) que si las cargas no se movieran entre sí. Eso crea una fuerza electrostática que en diferentes marcos de referencia puede considerarse una fuerza magnética. Pero aquí no contamos con la "ayuda" de esos cargos opuestos. Y creo que todavía ocurre el mismo efecto. ¿Cómo?

Lo siento, pero mis conocimientos de matemáticas y electromagnetismo no son tan extensos como para entender eso. Estaría bien si alguien pudiera explicar en términos sencillos debido a qué aspecto de la relatividad especial, tanto los observadores (en movimiento como estacionarios) ven desarrollarse los "mismos" eventos. Porque hasta donde yo sé, si ignoras el giro de las partículas, si dos partículas no se mueven de acuerdo con los marcos de referencia de las demás, no deberían sentir la fuerza magnética y, por lo tanto, no importa a qué velocidad viajen en comparación con, digamos, la tierra, nunca deberían sentir la fuerza de atracción.

Respuestas (4)

Respuesta corta:

La fuerza no es un invariante de Lorentz y tampoco lo es la aceleración. Los protones siempre se repelen entre sí, con una fuerza que combina las componentes eléctrica y magnética de la fuerza de Lorentz y depende del marco de referencia del observador, pero que se maximiza en su marco de reposo y se aproxima a cero a medida que los protones se vuelven ultra- relativista.

Detalles:

Exactamente su pregunta se trata en Purcell & Morin "Electricity & Magnetism" 3rd ed. pág.264.

El problema que puede tener es pensar que la fuerza es un invariante relativista, no lo es.

Los campos eléctricos y magnéticos se transforman al mirarlos desde un marco de referencia diferente.

En el marco estacionario de los protones, entonces solo existe la repulsión de Coulomb entre ellos dada por

F r mi s t = mi mi r mi s t = mi 2 4 π ϵ 0 r 2 r ^ ,
dónde mi r mi s t es el campo E de un protón estacionario.

En el marco de laboratorio, el campo eléctrico en la dirección entre los dos protones se incrementa por el factor de Lorentz a mi yo a b = γ mi r mi s t , dónde γ = ( 1 v 2 / C 2 ) 1 / 2 y donde γ 1 . Al mismo tiempo, hay un campo magnético causado por el movimiento de los protones y esto contribuye con una fuerza mi v × B yo a b , dónde B yo a b es el campo B medido en el marco de laboratorio.

El campo B de laboratorio se encuentra usando la transformada apropiada como

B yo a b = γ C 2 v × mi r mi s t

Por lo tanto, la fuerza entre los protones en el marco del laboratorio es

F yo a b = mi ( mi yo a b + v × B yo a b ) = mi ( γ mi r mi s t γ v 2 C 2 mi r mi s t ) = mi mi r mi s t γ = F r mi s t γ .
Esto es exactamente lo que requieren las reglas para transformar fuerzas bajo la relatividad especial. La fuerza que actúa entre los dos protones es menor en el marco del laboratorio y se aproxima a cero a medida que los protones se vuelven más y más relativistas.

Si lo dispuso de modo que sus haces de protones viajaran en líneas paralelas, también debe haber dispuesto que alguna fuerza actúe en la dirección opuesta a la repulsión mutua del protón. Esta fuerza se transformaría exactamente de la misma manera, de modo que si no hubiera aceleración neta en el sistema de laboratorio, tampoco habría aceleración neta en el sistema de reposo de protones.

"El problema que puede tener es pensar que la fuerza es un invariante relativista, no lo es". es engañoso: la velocidad relativa del observador no influye en la fuerza neta, cuánto se atribuye a E o B, que su siguiente línea dice correctamente: "Los campos eléctricos y magnéticos se transforman cuando se miran desde un marco de referencia diferente. "
Lea la respuesta completa @ f5r5e5d, la fuerza neta de hecho cambia por un factor de γ . ¿O está afirmando que las fuerzas son invariantes bajo las transformaciones de Lorentz? sciencebits.com/Transformation-Forces-Relativity . Es importante que aclaremos esto, ya que aquí hay varias respuestas confusas. ¿Leyó Purcell & Morin, o tiene otra fuente para su afirmación?
1er principio de la Relatividad Especial - Las leyes de la física son invariantes (es decir, idénticas) en todos los sistemas inerciales (marcos de referencia no acelerantes).
@f5r5e5d Y así son. La Ley F = q ( mi + v × B ) es cierto en ambos marcos de referencia. eso no quiere decir que F es invariante (y no lo es). De hecho, los vectores normales (como la fuerza, la aceleración, la velocidad, los campos eléctricos y magnéticos) NO PUEDEN ser invariantes bajo las transformaciones de Lorentz. Le sugiero que consulte cualquier libro de texto elemental sobre relatividad especial antes de publicar más comentarios que confundirán al OP.
"box car gedankenexperiment" - báscula de resorte que mide la fuerza entre los protones - la aguja no apunta a diferentes fuerzas para diferentes observadores
@ f5r5e5d Los vectores NO PUEDEN ser invariantes de Lorentz; solo los escalares y los 4 vectores pueden serlo. La fuerza opuesta aplicada por el resorte en la balanza de resorte se transforma EXACTAMENTE de la misma manera que la fuerza de Lorentz, por lo que la escala da exactamente la misma lectura. Sin embargo, la báscula solo tiene una calibración válida en el marco de reposo del instrumento. ¿Ya has mirado un libro de texto sobre fuerza en relatividad especial? Permítanme recomendar la sección 2 de core.ac.uk/download/pdf/11310772.pdf y la ecuación 19 en particular. En esta publicación tu X = 0 .
sí, miré, me equivoqué al suponer que "la física es la misma" equivale a que la fuerza neta es la misma. Sin embargo, la física es la misma , y ​​la aguja no se mueve para los observadores inerciales, que es el "panorama general" al que creo que no llegó en su respuesta: lo dejó colgado y abierto para confundir aún más el OP
@ f5r5e5d No, no es el panorama general en absoluto. El panorama general es que la fuerza no es un invariante de Lorentz y tampoco lo es la aceleración y que los protones se repelen entre sí con una fuerza diferente según el marco de referencia del observador. es decir, lo contrario de tres de sus comentarios.
Creo que esta conversación me ayudó, así que les daré las gracias a ambos. Según tengo entendido, a velocidades muy altas (marco de laboratorio), los protones se acelerarían entre sí debido a la enorme fuerza de atracción magnética, y si el marco de referencia viaja con los protones, se acelerarían entre sí debido a la fuerza electrostática. siendo la única fuerza que actúa sobre el movimiento ref. enmarca partículas estacionarias. Creo que esto no debería suceder ya que entonces 2 observadores no pudieron ponerse de acuerdo sobre si las partículas se están alejando o acercando entre sí.
@MaDrung En cuyo caso no has entendido nada. La fuerza es siempre repulsiva. En ninguna parte de mi respuesta o en ninguno de los comentarios anteriores, alguien sugiere que la fuerza pueda cambiar de signo.
@Rob jefferies: tal como lo entiendo ahora, con el aumento de la velocidad, aumentan tanto la fuerza electrostática como la magnética y ambas apuntan en direcciones opuestas. Mientras que la velocidad es 0, la fuerza magnética es 0. Cuando la velocidad aumenta, la fuerza magnética aumenta más que la fuerza electrostática, lo que significa que desde el punto de referencia del laboratorio, los protones se acelerarán entre sí más lentamente que si estuvieran estacionarios en el laboratorio. Esto podría pensarse para el observador estacionario en el laboratorio como una desaceleración del tiempo para las partículas en movimiento. Pero a la velocidad de la luz, la fuerza magnética y la electrostática se igualarían.
@Rob Jeffris ¿El hecho de que la fuerza no sea invariante de Lorenz sugiere que existe un marco en el que la fuerza de Lorenz sobre los protones es atracción? Puedo ver a partir de sus cálculos que la fuerza nunca puede ser atracción, pero ¿cómo puedo relacionar su signo que nunca cambia con su no invariancia?
@Simorq ¿No entiendo tu objeción? La fuerza entre dos protones siempre será repulsiva porque tienen una carga positiva (que es invariante).
@RobJeffries Pensé que la fuerza siempre sería repulsiva porque la fuerza magnética siempre sería menor que la fuerza eléctrica. Lo que estoy tratando de entender es por qué la fuerza magnética siempre es menor que la fuerza eléctrica, de modo que la fuerza neta siempre es repulsiva. Si la fuerza no es invariante de Lorenz, entonces podría haber un marco en el que la atracción magnética sea más fuerte que la repulsión eléctrica, ¿verdad? Pero no existe tal marco. ¿Por qué?
@Simorq Como no entiendo la base de su reclamo, no puedo refutarlo. Las matemáticas están ahí en mi respuesta, puedes ver que el término magnético nunca puede exceder el término eléctrico en amplitud porque v < C .
¿Tiene una referencia de libro electrónico para esta respuesta?

Siento que te estás perdiendo las fórmulas relativistas para los campos electromagnéticos esparcidos por cargas en movimiento uniforme. Estos se pueden encontrar como fórmulas (1538), (1539) en este enlace .

En este caso, la velocidad de las cargas en movimiento importa en ambos marcos en movimiento, por lo que debería obtener los mismos fenómenos físicos.

Pero, ¿cómo se puede determinar que se están moviendo? Relativamente entre sí no lo son. Por lo tanto, no deben sentir ninguna fuerza magnética y, por lo tanto, ninguna atracción. Pero si los mira desde un marco de referencia diferente, en realidad "chocan" juntos porque se mueven mucho más rápido que el observador y sienten los campos magnéticos de los demás. Algo no está bien con esto. Realmente no puedes decir a qué velocidad viaja algo a menos que estés calculando la velocidad relativa a otra cosa. Pero en un marco de referencia sienten una fuerza magnética atractiva, en otro no hay atracción.
Sigo pensando que aún no has hecho los cálculos con las fórmulas anteriores. La cuestión es que usted asume en el "marco del observador estacionario" que el campo eléctrico/magnético es lo mismo que una carga estática/una corriente uniforme e infinita que puede crear y eso no es cierto. Cuando ingresa la velocidad, en relación con el observador en ese marco, el equilibrio entre las fuerzas eléctricas y magnéticas debe ser el mismo que en el marco con el observador moviéndose junto con las dos partículas.
La fuerza no es un invariante relativista.
@MaDrung: supongamos que los protones se mueven cerca de la velocidad de la luz vista por un 1. observador. De acuerdo con las fórmulas anteriores, la repulsión de Colomb casi se cancela por la atracción magnética, por lo que ambos protones se moverán lentamente uno hacia el otro, porque la cancelación es casi completa. Ahora vas en el marco de descanso de los protones. Ves que ambas partículas se mueven rápidamente una hacia la otra debido solo a Colomb, ¿por qué? En el resto del sistema de los protones, el tiempo se dilata, fluye muy lentamente, y en estas unidades de tiempo que fluyen lentamente, de hecho, el movimiento lento visto por el primer observador parece finalmente rápido.
@Frederic Thomas: los protones estacionarios no se atraen, se repelen entre sí. Pero moviéndose en la misma dirección, lo hacen. Entonces, si se mueven muy rápido, digamos que superan la fuerza de Colomb, estarían acelerando uno hacia el otro, pero si su marco de referencia viaja con la velocidad horizontal de los protones, se están acelerando alejándose uno del otro. Este es el problema que quiero resolver. ¿Dónde cometí un error?
@RobJeffries gracias por su respuesta precisa anterior. No profundicé en las matemáticas como tú, pero lo que quería señalar es que el signo de la fuerza no cambia, pero no quise decir que el equilibrio de fuerzas debería ser exactamente el mismo.
@MaDrung: Lo siento por la confusión que creé. Le recomiendo las fórmulas dadas en las respuestas, tiene una fuerza de Colomb repelente entre los protones en el marco de referencia donde están en reposo (simplemente la ley de Colomb), y en un marco de referencia donde los protones se mueven perpendicularmente al dirección de la fuerza, la fuerza será más débil hasta que se cancele si los protones se movieron a la velocidad de la luz. Pero la fuerza magnética nunca será más fuerte que la fuerza de Colomb en cualquier sistema de referencia.
Gracias. Acabo de darme cuenta de eso ayer con un examen cuidadoso de las matemáticas y la ayuda de otros comentaristas. :) ¡Este sitio web es genial!

Ahora bien, esto se contradice por completo. ¿Tiene esto algo que ver con los efectos de la dilatación del tiempo y la contracción del espacio? Y si es así, ¿cómo?

Las fuerzas son diferentes en marcos diferentes, y eso no es un problema. Si la fuerza fuera cero en algún marco y distinta de cero en otro marco, eso sería un problema, pero afortunadamente hay una fuerza repulsiva entre esos dos protones en todos los marcos.

En el marco del par de protones, los protones tardan un tiempo en separarse una cierta distancia. En otro marco que el tiempo es más largo, este es el efecto de dilatación del tiempo. Si los protones se mueven muy rápido, entonces la fuerza entre ellos es muy pequeña y les toma mucho tiempo a los protones moverse una cierta distancia.

Entonces, ¿la fuerza electrostática es tan grande que, sin importar la velocidad de los protones, nunca sería superada por la fuerza magnética (que atrae en este caso)? ¿Y todos esos efectos se pueden explicar con la dilatación del tiempo?
"Entonces, la fuerza electrostática es tan grande que, sin importar la velocidad de los protones, nunca sería superada por la fuerza magnética (que atrae en este caso)". SÍ. "¿Y todos esos efectos se pueden explicar con la dilatación del tiempo?" NO. ... O QUIZÁS SÍ, YA QUE PODEMOS CALCULAR MUY FÁCILMENTE LA DISTANCIA ENTRE LOS PROTONES EN ALGÚN MOMENTO UTILIZANDO LA IDEA DE LA DILATACIÓN DEL TIEMPO. ... PERO LUEGO LO ÚNICO QUE SE EXPLICA ES EL MOVIMIENTO DE LOS PROTONES, SE EXPLICA POR UNA FUERZA DE COULOMB Y UN EFECTO DE DILATACIÓN DEL TIEMPO.

En el caso de 2 partículas cargadas iguales de carga mi (como el protón) en reposo en un marco de referencia, su fuerza mutua es, de acuerdo con la ley de Colomb, la conocida fórmula (en unidades SI): F = mi 2 4 π ϵ r 2

r es la distancia entre ambos, y ϵ la constante de dielectricidad del medio. Si ambas partículas se mueven con velocidad v si se observa desde otro marco de referencia, su fuerza (a continuación se consideran tanto la fuerza eléctrica como la magnética) es

F = mi 2 4 π ϵ r 2 ( 1 v 2 C 2 ) (a multiplicar por × γ , vea abajo)

dónde C es la velocidad de la luz. Si v = 0 se recupera la antigua ley de Colomb. Por otro lado, en caso de que ambos protones se muevan a la velocidad de la luz C la fuerza entre es cero. En otras palabras: a la velocidad de la luz, la fuerza eléctrica de Colomb y la fuerza magnética de Lorentz se cancelan entre sí (en este caso particular). Así es como funciona la electrodinámica. La contracción de la longitud solo ocurre en la dirección del movimiento que es perpendicular a la dirección de la fuerza, no juega ningún papel aquí. Podrías imaginar que debido a la dilatación del tiempo, que es infinita a la velocidad de la luz, que la repulsión está congelada, encaja con la visión intuitiva que uno podría tener sobre este efecto. Se puede encontrar una derivación de estas fórmulas en los libros de texto estándar, Jackson, por ejemplo.

Editar: Aparentemente, la segunda fórmula debe corregirse por un factor γ . Entonces resulta ser F = mi 2 4 π ϵ r 2 1 v 2 C 2 Sin embargo, la argumentación cualitativa no cambia.

¿Podría decir en qué parte de Jackson se deriva esto?
Inserte la ecuación (11.152) en (12.1). Entonces lo entiendes.
Me temo que no.
De Jackson no es tan evidente. Le recomiendo que busque en Google "carga espacial de haces de protones", encontrará notas de conferencias que lo explicarán.
Aquí hay un artículo del CERN sobre la carga espacial: acceleratorinstitute.web.cern.ch/acceleratorinstitute/ACINST89/…
No, lo que quiero decir es que simplemente te equivocaste por un factor de γ . Y también atribuye algo a una referencia que no dice, que es peor.
Veo que hay una discrepancia de uno γ . Por el momento no veo de donde viene, en la lección que cité arriba el resultado sale con un factor 1 γ 2 .
Pero esa referencia no dice que la fuerza en el marco de apoyo de la viga se modifica por ese factor. La densidad de carga en el marco de reposo de la partícula (y por lo tanto la fuerza del marco de reposo) también debe modificarse (reducirse) debido a la contracción de la longitud.
Mientras tanto, busqué alguna otra fuente y parece ser que su fórmula es correcta. El argumento con la contracción de la longitud de la densidad de la línea aparentemente explica la 1 / γ discrepancia.